Fair gambler's ruin problem intuitionProbability of Gambler's Ruin with Unequal Gain/LossAdaptive gambler's...

Do creatures with a listed speed of "0 ft., fly 30 ft. (hover)" ever touch the ground?

Sums of two squares in arithmetic progressions

Send out email when Apex Queueable fails and test it

Placement of More Information/Help Icon button for Radio Buttons

Notepad++ delete until colon for every line with replace all

How can saying a song's name be a copyright violation?

Why is the sentence "Das ist eine Nase" correct?

How to Prove P(a) → ∀x(P(x) ∨ ¬(x = a)) using Natural Deduction

Why was Sir Cadogan fired?

If a warlock makes a Dancing Sword their pact weapon, is there a way to prevent it from disappearing if it's farther away for more than a minute?

What is a Samsaran Word™?

Why were 5.25" floppy drives cheaper than 8"?

Finitely generated matrix groups whose eigenvalues are all algebraic

What historical events would have to change in order to make 19th century "steampunk" technology possible?

How do conventional missiles fly?

Theorists sure want true answers to this!

files created then deleted at every second in tmp directory

How can I deal with my CEO asking me to hire someone with a higher salary than me, a co-founder?

What reasons are there for a Capitalist to oppose a 100% inheritance tax?

GFCI outlets - can they be repaired? Are they really needed at the end of a circuit?

Finding the reason behind the value of the integral.

How many wives did king shaul have

Ambiguity in the definition of entropy

OP Amp not amplifying audio signal



Fair gambler's ruin problem intuition


Probability of Gambler's Ruin with Unequal Gain/LossAdaptive gambler's ruin problemGambler's Ruin with no set target for winGambler's ruin problem - unsure about the number of roundsEffect of Gambler's Ruin Bet Size on DurationGambler's ruin: verifying Markov propertyComparison of duration of two gambler's ruin gamesGambler's Ruin - Probability of Losing in t StepsGambler's Ruin: Win 2 dollars, Lose 1 dollarGambler's ruin Markov chain













1












$begingroup$


In a fair gambler's ruin problem, where the gambler starts with k dollars, wins $1 with probability 1/2 and loses $1 with probability 1/2, and stops when he/she reaches $n or $0.



In the solution (from Dobrow's Introduction to Stochastic Processes with R), they let $p_k$ be defined as the probability of reaching $n with $k in one's inventory. Then they use the fact that $p_k - p_{k-1} = p_{k-1} - p_{k-2} = ... = p_1 - p_0 = p_1$.



Intuitively this means the probability of reaching $n with $k minus the probability of reaching $n with $k-1 is equivalent to the probability of reaching $n with only $1.



Is there an intuitive reason why this is the case?










share|cite









$endgroup$

















    1












    $begingroup$


    In a fair gambler's ruin problem, where the gambler starts with k dollars, wins $1 with probability 1/2 and loses $1 with probability 1/2, and stops when he/she reaches $n or $0.



    In the solution (from Dobrow's Introduction to Stochastic Processes with R), they let $p_k$ be defined as the probability of reaching $n with $k in one's inventory. Then they use the fact that $p_k - p_{k-1} = p_{k-1} - p_{k-2} = ... = p_1 - p_0 = p_1$.



    Intuitively this means the probability of reaching $n with $k minus the probability of reaching $n with $k-1 is equivalent to the probability of reaching $n with only $1.



    Is there an intuitive reason why this is the case?










    share|cite









    $endgroup$















      1












      1








      1





      $begingroup$


      In a fair gambler's ruin problem, where the gambler starts with k dollars, wins $1 with probability 1/2 and loses $1 with probability 1/2, and stops when he/she reaches $n or $0.



      In the solution (from Dobrow's Introduction to Stochastic Processes with R), they let $p_k$ be defined as the probability of reaching $n with $k in one's inventory. Then they use the fact that $p_k - p_{k-1} = p_{k-1} - p_{k-2} = ... = p_1 - p_0 = p_1$.



      Intuitively this means the probability of reaching $n with $k minus the probability of reaching $n with $k-1 is equivalent to the probability of reaching $n with only $1.



      Is there an intuitive reason why this is the case?










      share|cite









      $endgroup$




      In a fair gambler's ruin problem, where the gambler starts with k dollars, wins $1 with probability 1/2 and loses $1 with probability 1/2, and stops when he/she reaches $n or $0.



      In the solution (from Dobrow's Introduction to Stochastic Processes with R), they let $p_k$ be defined as the probability of reaching $n with $k in one's inventory. Then they use the fact that $p_k - p_{k-1} = p_{k-1} - p_{k-2} = ... = p_1 - p_0 = p_1$.



      Intuitively this means the probability of reaching $n with $k minus the probability of reaching $n with $k-1 is equivalent to the probability of reaching $n with only $1.



      Is there an intuitive reason why this is the case?







      probability






      share|cite













      share|cite











      share|cite




      share|cite










      asked 3 hours ago









      platypus17platypus17

      366




      366






















          2 Answers
          2






          active

          oldest

          votes


















          2












          $begingroup$

          Regarding an "intuitive" reason for this relation, note that winning or losing a dollar has an equal chance and is independent of how much your currently have. Thus, the change in probability of winning or losing when starting off with $$1$ more is independent of what your starting value is. Note that if $q_k = 1 - p_k$ is the probability of losing when starting with $$k$, then plugging $p_k = 1 - q_k$ in gives that



          $$q_{k-1} - q_k = q_{k-2} - q_{k - 1} = ldots = q_1 - q_2 = q_0 - q_1 tag{1}label{eq1}$$



          Note you can reverse all the elements by multiplying by $-1$ to give the exact same relationship as with $p_k$.



          Regarding how to get the relationship, this answer originally started with that, as the answer by John Doe states, the difference relation for reaching $n starting with $i is given by



          $$p_i = frac{1}{2}p_{i - 1} + frac{1}{2}p_{i + 1} tag{2}label{eq2}$$



          based on the probabilities of either winning or losing the first time. Summing eqref{eq2} for $i$ from $1$ to $k - 1$ gives



          $$sum_{i=1}^{k-1} p_i = frac{1}{2}sum_{i=1}^{k-1} p_{i - 1} + frac{1}{2}sum_{i=1}^{k-1} p_{i + 1} tag{3}label{eq3}$$



          Having the summations only include the common terms on both sides gives



          $$p_1 + sum_{i=2}^{k - 2} p_i + p_{k-1} = frac{1}{2}p_0 + frac{1}{2}p_1 + frac{1}{2}sum_{i=2}^{k - 2} p_i + frac{1}{2}sum_{i=2}^{k - 2} p_i + frac{1}{2}p_{k-1} + frac{1}{2}p_k tag{4}label{eq4}$$



          Since the summation parts on both sides up to the same thing, they can be removed. Thus, after moving the $p_0$ and $p_1$ terms to the LHS and the $p_{k-1}$ term on the left to the RHS, eqref{eq4} becomes



          $$frac{1}{2}p_1 - frac{1}{2}p_0 = frac{1}{2}p_k - frac{1}{2}p_{k-1} tag{5}label{eq5}$$



          Multiplying both sides by $2$, then varying $k$ down, gives the relations you stated are used in the solution. However, it's generally simpler & easier to just manipulate eqref{eq2} to get that $p_{i+1} - p_{i} = p_{i} - p_{i-1}$, like John Doe's answer states.






          share|cite|improve this answer











          $endgroup$





















            3












            $begingroup$

            The probability of reaching $n staring with $k can be split up by what possible first steps you can take - you either lose the first toss or win, each with probability 1/2. Then $$p_k=frac12(p_{k-1}+p_{k+1})$$ Rearranging this gives $$2p_k=p_{k-1}+p_{k+1}\p_k-p_{k-1}=p_{k+1}-p_k$$ as required, and iterating it multiple times gets to $p_1-p_0$, and of course, $p_0=0$.






            share|cite|improve this answer











            $endgroup$














              Your Answer





              StackExchange.ifUsing("editor", function () {
              return StackExchange.using("mathjaxEditing", function () {
              StackExchange.MarkdownEditor.creationCallbacks.add(function (editor, postfix) {
              StackExchange.mathjaxEditing.prepareWmdForMathJax(editor, postfix, [["$", "$"], ["\\(","\\)"]]);
              });
              });
              }, "mathjax-editing");

              StackExchange.ready(function() {
              var channelOptions = {
              tags: "".split(" "),
              id: "69"
              };
              initTagRenderer("".split(" "), "".split(" "), channelOptions);

              StackExchange.using("externalEditor", function() {
              // Have to fire editor after snippets, if snippets enabled
              if (StackExchange.settings.snippets.snippetsEnabled) {
              StackExchange.using("snippets", function() {
              createEditor();
              });
              }
              else {
              createEditor();
              }
              });

              function createEditor() {
              StackExchange.prepareEditor({
              heartbeatType: 'answer',
              autoActivateHeartbeat: false,
              convertImagesToLinks: true,
              noModals: true,
              showLowRepImageUploadWarning: true,
              reputationToPostImages: 10,
              bindNavPrevention: true,
              postfix: "",
              imageUploader: {
              brandingHtml: "Powered by u003ca class="icon-imgur-white" href="https://imgur.com/"u003eu003c/au003e",
              contentPolicyHtml: "User contributions licensed under u003ca href="https://creativecommons.org/licenses/by-sa/3.0/"u003ecc by-sa 3.0 with attribution requiredu003c/au003e u003ca href="https://stackoverflow.com/legal/content-policy"u003e(content policy)u003c/au003e",
              allowUrls: true
              },
              noCode: true, onDemand: true,
              discardSelector: ".discard-answer"
              ,immediatelyShowMarkdownHelp:true
              });


              }
              });














              draft saved

              draft discarded


















              StackExchange.ready(
              function () {
              StackExchange.openid.initPostLogin('.new-post-login', 'https%3a%2f%2fmath.stackexchange.com%2fquestions%2f3172677%2ffair-gamblers-ruin-problem-intuition%23new-answer', 'question_page');
              }
              );

              Post as a guest















              Required, but never shown

























              2 Answers
              2






              active

              oldest

              votes








              2 Answers
              2






              active

              oldest

              votes









              active

              oldest

              votes






              active

              oldest

              votes









              2












              $begingroup$

              Regarding an "intuitive" reason for this relation, note that winning or losing a dollar has an equal chance and is independent of how much your currently have. Thus, the change in probability of winning or losing when starting off with $$1$ more is independent of what your starting value is. Note that if $q_k = 1 - p_k$ is the probability of losing when starting with $$k$, then plugging $p_k = 1 - q_k$ in gives that



              $$q_{k-1} - q_k = q_{k-2} - q_{k - 1} = ldots = q_1 - q_2 = q_0 - q_1 tag{1}label{eq1}$$



              Note you can reverse all the elements by multiplying by $-1$ to give the exact same relationship as with $p_k$.



              Regarding how to get the relationship, this answer originally started with that, as the answer by John Doe states, the difference relation for reaching $n starting with $i is given by



              $$p_i = frac{1}{2}p_{i - 1} + frac{1}{2}p_{i + 1} tag{2}label{eq2}$$



              based on the probabilities of either winning or losing the first time. Summing eqref{eq2} for $i$ from $1$ to $k - 1$ gives



              $$sum_{i=1}^{k-1} p_i = frac{1}{2}sum_{i=1}^{k-1} p_{i - 1} + frac{1}{2}sum_{i=1}^{k-1} p_{i + 1} tag{3}label{eq3}$$



              Having the summations only include the common terms on both sides gives



              $$p_1 + sum_{i=2}^{k - 2} p_i + p_{k-1} = frac{1}{2}p_0 + frac{1}{2}p_1 + frac{1}{2}sum_{i=2}^{k - 2} p_i + frac{1}{2}sum_{i=2}^{k - 2} p_i + frac{1}{2}p_{k-1} + frac{1}{2}p_k tag{4}label{eq4}$$



              Since the summation parts on both sides up to the same thing, they can be removed. Thus, after moving the $p_0$ and $p_1$ terms to the LHS and the $p_{k-1}$ term on the left to the RHS, eqref{eq4} becomes



              $$frac{1}{2}p_1 - frac{1}{2}p_0 = frac{1}{2}p_k - frac{1}{2}p_{k-1} tag{5}label{eq5}$$



              Multiplying both sides by $2$, then varying $k$ down, gives the relations you stated are used in the solution. However, it's generally simpler & easier to just manipulate eqref{eq2} to get that $p_{i+1} - p_{i} = p_{i} - p_{i-1}$, like John Doe's answer states.






              share|cite|improve this answer











              $endgroup$


















                2












                $begingroup$

                Regarding an "intuitive" reason for this relation, note that winning or losing a dollar has an equal chance and is independent of how much your currently have. Thus, the change in probability of winning or losing when starting off with $$1$ more is independent of what your starting value is. Note that if $q_k = 1 - p_k$ is the probability of losing when starting with $$k$, then plugging $p_k = 1 - q_k$ in gives that



                $$q_{k-1} - q_k = q_{k-2} - q_{k - 1} = ldots = q_1 - q_2 = q_0 - q_1 tag{1}label{eq1}$$



                Note you can reverse all the elements by multiplying by $-1$ to give the exact same relationship as with $p_k$.



                Regarding how to get the relationship, this answer originally started with that, as the answer by John Doe states, the difference relation for reaching $n starting with $i is given by



                $$p_i = frac{1}{2}p_{i - 1} + frac{1}{2}p_{i + 1} tag{2}label{eq2}$$



                based on the probabilities of either winning or losing the first time. Summing eqref{eq2} for $i$ from $1$ to $k - 1$ gives



                $$sum_{i=1}^{k-1} p_i = frac{1}{2}sum_{i=1}^{k-1} p_{i - 1} + frac{1}{2}sum_{i=1}^{k-1} p_{i + 1} tag{3}label{eq3}$$



                Having the summations only include the common terms on both sides gives



                $$p_1 + sum_{i=2}^{k - 2} p_i + p_{k-1} = frac{1}{2}p_0 + frac{1}{2}p_1 + frac{1}{2}sum_{i=2}^{k - 2} p_i + frac{1}{2}sum_{i=2}^{k - 2} p_i + frac{1}{2}p_{k-1} + frac{1}{2}p_k tag{4}label{eq4}$$



                Since the summation parts on both sides up to the same thing, they can be removed. Thus, after moving the $p_0$ and $p_1$ terms to the LHS and the $p_{k-1}$ term on the left to the RHS, eqref{eq4} becomes



                $$frac{1}{2}p_1 - frac{1}{2}p_0 = frac{1}{2}p_k - frac{1}{2}p_{k-1} tag{5}label{eq5}$$



                Multiplying both sides by $2$, then varying $k$ down, gives the relations you stated are used in the solution. However, it's generally simpler & easier to just manipulate eqref{eq2} to get that $p_{i+1} - p_{i} = p_{i} - p_{i-1}$, like John Doe's answer states.






                share|cite|improve this answer











                $endgroup$
















                  2












                  2








                  2





                  $begingroup$

                  Regarding an "intuitive" reason for this relation, note that winning or losing a dollar has an equal chance and is independent of how much your currently have. Thus, the change in probability of winning or losing when starting off with $$1$ more is independent of what your starting value is. Note that if $q_k = 1 - p_k$ is the probability of losing when starting with $$k$, then plugging $p_k = 1 - q_k$ in gives that



                  $$q_{k-1} - q_k = q_{k-2} - q_{k - 1} = ldots = q_1 - q_2 = q_0 - q_1 tag{1}label{eq1}$$



                  Note you can reverse all the elements by multiplying by $-1$ to give the exact same relationship as with $p_k$.



                  Regarding how to get the relationship, this answer originally started with that, as the answer by John Doe states, the difference relation for reaching $n starting with $i is given by



                  $$p_i = frac{1}{2}p_{i - 1} + frac{1}{2}p_{i + 1} tag{2}label{eq2}$$



                  based on the probabilities of either winning or losing the first time. Summing eqref{eq2} for $i$ from $1$ to $k - 1$ gives



                  $$sum_{i=1}^{k-1} p_i = frac{1}{2}sum_{i=1}^{k-1} p_{i - 1} + frac{1}{2}sum_{i=1}^{k-1} p_{i + 1} tag{3}label{eq3}$$



                  Having the summations only include the common terms on both sides gives



                  $$p_1 + sum_{i=2}^{k - 2} p_i + p_{k-1} = frac{1}{2}p_0 + frac{1}{2}p_1 + frac{1}{2}sum_{i=2}^{k - 2} p_i + frac{1}{2}sum_{i=2}^{k - 2} p_i + frac{1}{2}p_{k-1} + frac{1}{2}p_k tag{4}label{eq4}$$



                  Since the summation parts on both sides up to the same thing, they can be removed. Thus, after moving the $p_0$ and $p_1$ terms to the LHS and the $p_{k-1}$ term on the left to the RHS, eqref{eq4} becomes



                  $$frac{1}{2}p_1 - frac{1}{2}p_0 = frac{1}{2}p_k - frac{1}{2}p_{k-1} tag{5}label{eq5}$$



                  Multiplying both sides by $2$, then varying $k$ down, gives the relations you stated are used in the solution. However, it's generally simpler & easier to just manipulate eqref{eq2} to get that $p_{i+1} - p_{i} = p_{i} - p_{i-1}$, like John Doe's answer states.






                  share|cite|improve this answer











                  $endgroup$



                  Regarding an "intuitive" reason for this relation, note that winning or losing a dollar has an equal chance and is independent of how much your currently have. Thus, the change in probability of winning or losing when starting off with $$1$ more is independent of what your starting value is. Note that if $q_k = 1 - p_k$ is the probability of losing when starting with $$k$, then plugging $p_k = 1 - q_k$ in gives that



                  $$q_{k-1} - q_k = q_{k-2} - q_{k - 1} = ldots = q_1 - q_2 = q_0 - q_1 tag{1}label{eq1}$$



                  Note you can reverse all the elements by multiplying by $-1$ to give the exact same relationship as with $p_k$.



                  Regarding how to get the relationship, this answer originally started with that, as the answer by John Doe states, the difference relation for reaching $n starting with $i is given by



                  $$p_i = frac{1}{2}p_{i - 1} + frac{1}{2}p_{i + 1} tag{2}label{eq2}$$



                  based on the probabilities of either winning or losing the first time. Summing eqref{eq2} for $i$ from $1$ to $k - 1$ gives



                  $$sum_{i=1}^{k-1} p_i = frac{1}{2}sum_{i=1}^{k-1} p_{i - 1} + frac{1}{2}sum_{i=1}^{k-1} p_{i + 1} tag{3}label{eq3}$$



                  Having the summations only include the common terms on both sides gives



                  $$p_1 + sum_{i=2}^{k - 2} p_i + p_{k-1} = frac{1}{2}p_0 + frac{1}{2}p_1 + frac{1}{2}sum_{i=2}^{k - 2} p_i + frac{1}{2}sum_{i=2}^{k - 2} p_i + frac{1}{2}p_{k-1} + frac{1}{2}p_k tag{4}label{eq4}$$



                  Since the summation parts on both sides up to the same thing, they can be removed. Thus, after moving the $p_0$ and $p_1$ terms to the LHS and the $p_{k-1}$ term on the left to the RHS, eqref{eq4} becomes



                  $$frac{1}{2}p_1 - frac{1}{2}p_0 = frac{1}{2}p_k - frac{1}{2}p_{k-1} tag{5}label{eq5}$$



                  Multiplying both sides by $2$, then varying $k$ down, gives the relations you stated are used in the solution. However, it's generally simpler & easier to just manipulate eqref{eq2} to get that $p_{i+1} - p_{i} = p_{i} - p_{i-1}$, like John Doe's answer states.







                  share|cite|improve this answer














                  share|cite|improve this answer



                  share|cite|improve this answer








                  edited 1 hour ago

























                  answered 2 hours ago









                  John OmielanJohn Omielan

                  4,5362215




                  4,5362215























                      3












                      $begingroup$

                      The probability of reaching $n staring with $k can be split up by what possible first steps you can take - you either lose the first toss or win, each with probability 1/2. Then $$p_k=frac12(p_{k-1}+p_{k+1})$$ Rearranging this gives $$2p_k=p_{k-1}+p_{k+1}\p_k-p_{k-1}=p_{k+1}-p_k$$ as required, and iterating it multiple times gets to $p_1-p_0$, and of course, $p_0=0$.






                      share|cite|improve this answer











                      $endgroup$


















                        3












                        $begingroup$

                        The probability of reaching $n staring with $k can be split up by what possible first steps you can take - you either lose the first toss or win, each with probability 1/2. Then $$p_k=frac12(p_{k-1}+p_{k+1})$$ Rearranging this gives $$2p_k=p_{k-1}+p_{k+1}\p_k-p_{k-1}=p_{k+1}-p_k$$ as required, and iterating it multiple times gets to $p_1-p_0$, and of course, $p_0=0$.






                        share|cite|improve this answer











                        $endgroup$
















                          3












                          3








                          3





                          $begingroup$

                          The probability of reaching $n staring with $k can be split up by what possible first steps you can take - you either lose the first toss or win, each with probability 1/2. Then $$p_k=frac12(p_{k-1}+p_{k+1})$$ Rearranging this gives $$2p_k=p_{k-1}+p_{k+1}\p_k-p_{k-1}=p_{k+1}-p_k$$ as required, and iterating it multiple times gets to $p_1-p_0$, and of course, $p_0=0$.






                          share|cite|improve this answer











                          $endgroup$



                          The probability of reaching $n staring with $k can be split up by what possible first steps you can take - you either lose the first toss or win, each with probability 1/2. Then $$p_k=frac12(p_{k-1}+p_{k+1})$$ Rearranging this gives $$2p_k=p_{k-1}+p_{k+1}\p_k-p_{k-1}=p_{k+1}-p_k$$ as required, and iterating it multiple times gets to $p_1-p_0$, and of course, $p_0=0$.







                          share|cite|improve this answer














                          share|cite|improve this answer



                          share|cite|improve this answer








                          edited 1 hour ago

























                          answered 2 hours ago









                          John DoeJohn Doe

                          11.5k11239




                          11.5k11239






























                              draft saved

                              draft discarded




















































                              Thanks for contributing an answer to Mathematics Stack Exchange!


                              • Please be sure to answer the question. Provide details and share your research!

                              But avoid



                              • Asking for help, clarification, or responding to other answers.

                              • Making statements based on opinion; back them up with references or personal experience.


                              Use MathJax to format equations. MathJax reference.


                              To learn more, see our tips on writing great answers.




                              draft saved


                              draft discarded














                              StackExchange.ready(
                              function () {
                              StackExchange.openid.initPostLogin('.new-post-login', 'https%3a%2f%2fmath.stackexchange.com%2fquestions%2f3172677%2ffair-gamblers-ruin-problem-intuition%23new-answer', 'question_page');
                              }
                              );

                              Post as a guest















                              Required, but never shown





















































                              Required, but never shown














                              Required, but never shown












                              Required, but never shown







                              Required, but never shown

































                              Required, but never shown














                              Required, but never shown












                              Required, but never shown







                              Required, but never shown







                              Popular posts from this blog

                              Paper upload error, “Upload failed: The top margin is 0.715 in on page 3, which is below the required...

                              Emraan Hashmi Filmografia | Linki zewnętrzne | Menu nawigacyjneGulshan GroverGulshan...

                              How can I write this formula?newline and italics added with leqWhy does widehat behave differently if I...